Đến nội dung

Hình ảnh

ÔN THI ĐẠI HỌC 2012


  • Please log in to reply
Chủ đề này có 95 trả lời

#41
lehoanghiep

lehoanghiep

    Trung sĩ

  • Thành viên
  • 196 Bài viết

Bài 28: Cho $x,y,z$ thực thỏa mãn $x+y+z=0$. Tìm min $Q=|2x-y|+|2y-z|+|2z-x|-\ln(\sqrt{14(x^2+y^2+z^2)}+1)$
Bài này khá giống Câu 6 khối A 2012

Đặt $a=2x-y;b=2y-z;c=2z-x$. Ta có $a+b+c=0$ và $a^{2}+b^{2}+c^{2}=5\left ( x^{2}+y^{2}+z^{2} \right )-4\left ( xy+yz+zx \right )=7\left ( x^{2}+y^{2}+z^{2} \right )$ (vì $x+y+z=0$).
Khi đó $Q=\left | a \right |+\left | b \right |+\left | c \right |-ln\left ( \sqrt{2\left ( a^{2}+b^{2}+c^{2} \right )} +1\right )$.
Mặt khác $a^{2}+b^{2}+c^{2}=-2\left ( ab+bc+ca \right )\leq 2\left ( \left | ab \right |+\left | bc \right |+\left | ca \right | \right )$ nên
$\left ( \left | a \right |+\left | b \right |+\left | c \right | \right )^{2}\geq 2\left ( a^{2}+b^{2}+c^{2} \right )$.
Suy ra $Q\geq t-ln\left ( t+1 \right )=f\left ( t \right )$ với $t=\left | a \right |+\left | b \right |+\left | c \right |\geq 0$.
$f'\left ( t \right )=\frac{1}{t+1}>0\Rightarrow Q\geq f\left ( 0 \right )=0$.
Vậy $Q_{min}=0$. Đẳng thức xảy ra khi và chỉ khi $x=y=z=0$.

#42
le_hoang1995

le_hoang1995

    Sĩ quan

  • Thành viên
  • 314 Bài viết

Bài 30:
Ch0 các số thực $a,b$ và số thực dương $c$ thỏa mãn $a^2+b^2+ab=3c^2$.Chứng minh rằng:
$$a^3+b^3+4abc\leq 6c^3$$

Giả thiết tương đương $a^2c+b^2c+abc=3c^3$
$3c^2=(a+b)^2-ab\geq (a+b)^2-\frac{(a+b)^2}{4}=\frac{3(a+b)^2}{4}\Leftrightarrow 2c\geq a+b$
BĐT cần chứng minh tương đương với
$a^3+b^3+4abc\leq 2a^2c+2b^2c+2abc$
$\Leftrightarrow (a+b)(a^2+b^2-ab)+2abc\leq 2c(a^2+b^2)$
$\Leftrightarrow (a+b)(a^2+b^2-ab)-2c(a^2+b^2-ab)\leq 0$
$\Leftrightarrow (a^2+b^2-ab)(a+b-2c)\leq 0$
Đúng theo điều kiện. Dấu bằng xảy ra khi $a=b=c=1$
____________________________________________

Bài 31 Cho $0<y<x \leq 3$ và $x+y \leq 5$, tìm GTLN của các biểu thức
$S_2=x^2+y^2\\ S_3=x^3+y^3\\ S_n=x^n+y^n$

Bài viết đã được chỉnh sửa nội dung bởi le_hoang1995: 31-10-2012 - 15:37


#43
lehoanghiep

lehoanghiep

    Trung sĩ

  • Thành viên
  • 196 Bài viết
Bài 32 (Thi thử THPT Chuyên ĐH Vinh lần 2-2012)
Cho các số thực $a,b,c$ thuộc đoạn $\left [ 0;1 \right ]$. Tìm giá trị lớn nhất của biểu thức $P=\frac{a^{3}+2}{b^{2}+1}+\frac{b^{3}+2}{c^{2}+1}+\frac{c^{3}+2}{a^{2}+1}$.

Bài viết đã được chỉnh sửa nội dung bởi WhjteShadow: 05-10-2012 - 19:43


#44
lehoanghiep

lehoanghiep

    Trung sĩ

  • Thành viên
  • 196 Bài viết
Bài 33 (Thi thử THPT Chuyên ĐH Vinh lần 2-2010)
Cho các số thực không âm $x,y,z$ thỏa mãn $xy+yz+zx=3$. Tìm giá trị nhỏ nhất của biểu thức
$A=x^{2}y^{3}+y^{2}z^{3}+z^{2}x^{3}+\left ( x-1 \right )^{2}+\left ( y-1 \right )^{2}+\left ( z-1 \right )^{2}$.

#45
Ispectorgadget

Ispectorgadget

    Nothing

  • Quản lý Toán Phổ thông
  • 2946 Bài viết

Bài 31 Cho $0<y<x \leq 3$ và $x+y \leq 5$, tìm GTLN của các biểu thức
$S_2=x^2+y^2\\ S_3=x^3+y^3\\ S_n=x^n+y^n$

Tính trước cái tổng $S_2=x^2+y^2$
Sử dụng phép nhóm Abel ta có $S_2=x.x+y.y=x(x-y)+y(x+y)\le 3(x-y)+5y=3x+2y\le 13$
Dấu "=" xảy ra khi $x=3;y=2$

Bài viết đã được chỉnh sửa nội dung bởi Ispectorgadget: 05-10-2012 - 23:19

►|| The aim of life is self-development. To realize one's nature perfectly - that is what each of us is here for. ™ ♫


#46
lehoanghiep

lehoanghiep

    Trung sĩ

  • Thành viên
  • 196 Bài viết

Bài 25: Cho $a,b,c$ thực dương thỏa $\sqrt{a^2+b^2}+\sqrt{b^2+c^2}+\sqrt{c^2+a^2}\le 3\sqrt{2}$. Chứng minh $$\frac{1}{\sqrt{8^a+1}}+\frac{1}{\sqrt{8^b+1}}+\frac{1}{\sqrt{8^c+1}}\ge 1$$
Trường THPT Gia Tĩnh Thanh Hóa 2012

Từ giả thiết, áp dụng BĐT $x^{2}+y^{2}\geq \frac{\left ( x+y \right )^{2}}{2}$ ta được $a+b+c\leq 3$.
Đặt $2^{a}=x;2^{b}=y;2^{c}=z$. Khi đó $xyz\leq 8$.
$VT=\frac{1}{\sqrt{x^{3}+1}}+\frac{1}{\sqrt{y^{3}+1}}+\frac{1}{\sqrt{z^{3}+1}}$.
Áp dụng BĐT Cauchy, ta có $\sqrt{x^{3}+1}=\sqrt{\left ( x+1 \right )\left ( x^{2}-x+1 \right )}\leq \frac{x^{2}+2}{2}$.
BĐT cần chứng minh đúng nếu ta chứng minh được $\frac{2}{x^{2}+2}+\frac{2}{y^{2}+2}+\frac{2}{z^{2}+2}\geq 1$
$\Leftrightarrow 4\left ( x^{2}+y^{2}+z^{2} \right )+16\geq x^{2}y^{2}z^{2}$. (*)
Mà $4\left ( x^{2}+y^{2}+z^{2} \right )\geq 12\sqrt[3]{x^{2}y^{2}z^{2}}\geq6xyz\geq \frac{3x^{2}y^{2}z^{2}}{4}$ (vì $xyz\leq 8$).
Do đó (*) đúng. Suy ra đpcm. Đẳng thức xảy ra khi và chỉ khi $a=b=c=1$.

Bài viết đã được chỉnh sửa nội dung bởi lehoanghiep: 05-10-2012 - 23:30


#47
tim1nuathatlac

tim1nuathatlac

    Thượng sĩ

  • Thành viên
  • 298 Bài viết
Bài 34 Cho $a,b,c\in \left [ 0;1 \right ]$ và $a+b+c\neq 0$. CMR $\frac{1}{ab+1}+\frac{1}{bc+1}+\frac{1}{ca+1}\leq \frac{5}{a+b+c}$

Bài 35 Cho $a,b,c$ là 3 cạnh của tam giác, CMR $\frac{a+c}{3a+b}+\frac{a+b}{3a+c}+\frac{2a}{2a+b+c}< 2$

Gợi ý $x=\frac{a+b}{2};y=\frac{a+c}{2};z=a\Rightarrow x+y> z;y+z> x;z+x> y$


#48
lehoanghiep

lehoanghiep

    Trung sĩ

  • Thành viên
  • 196 Bài viết

Bài 34 Cho $a,b,c\in \left [ 0;1 \right ]$ và $a+b+c\neq 0$. CMR $\frac{1}{ab+1}+\frac{1}{bc+1}+\frac{1}{ca+1}\leq \frac{5}{a+b+c}$

Ta có $\left ( 1-a \right )\left ( 1-b \right )\geq 0\Rightarrow ab+1\geq a+b$. Tương tự $bc+1\geq b+c;ca+1\geq c+a$.
Khi đó $\left ( a+b+c \right )\left ( \frac{1}{ab+1}+\frac{1}{bc+1}+\frac{1}{ca+1} \right )\leq \frac{a}{bc+1}+\frac{b}{ca+1}+\frac{c}{ab+1}+3$
$\leq a\left ( \frac{1}{bc+1}-\frac{c}{ca+b}-\frac{b}{ab+c} \right )+5\leq a\left ( 1-\frac{c}{c+b}-\frac{b}{b+c} \right )+5\leq 5$.
Vậy ta có đpcm. Đẳng thức xảy ra khi và chỉ khi $a=b=c=1$.

Bài viết đã được chỉnh sửa nội dung bởi lehoanghiep: 07-10-2012 - 09:59


#49
WhjteShadow

WhjteShadow

    Thượng úy

  • Phó Quản lý Toán Ứng dụ
  • 1323 Bài viết

Bài 32 (Thi thử THPT Chuyên ĐH Vinh lần 2-2012)
Cho các số thực $a,b,c$ thuộc đoạn $\left [ 0;1 \right ]$. Tìm giá trị lớn nhất của biểu thức $P=\frac{a^{3}+2}{b^{2}+1}+\frac{b^{3}+2}{c^{2}+1}+\frac{c^{3}+2}{a^{2}+1}$.

Đặt $P(a;b;c)=\frac{a^{3}+2}{b^{2}+1}+\frac{b^{3}+2}{c^{2}+1}+\frac{c^{3}+2}{a^{2}+1}$
Ta sẽ chứng minh $P(a;b;c)\leq P(a;b;0)$ thật vậy điều đó tương đương:
$$\frac{b^{3}+2}{c^{2}+1}+\frac{c^{3}+2}{a^{2}+1}\leq b^2+2+\frac{2}{a^2+1}$$
$$\Leftrightarrow (b^3+2)\left(1-\frac{1}{c^2+1}\right)-\frac{c^3}{a^2+1}\geq 0$$
$$\Leftrightarrow c^2\left(\frac{(b^3+2)(a^2+1)-a^3-c}{(a^2+1)(c^2+1)}\right)\geq 0$$
Và điều này luôn đúng do $(b^3+2)(a^2+1)\geq 2\geq c^3+c$
Tương tự vậy ta dễ dàng chứng minh $P(a;b;c)\leq P(a;b;0)\leq P(a;0;0)\leq P(0;0;0)=6$
Vậy giá trị lớn nhất của $P=6$.Đạt được khi $a=b=c=0$ $\square$

Bài viết đã được chỉnh sửa nội dung bởi WhjteShadow: 06-10-2012 - 12:39

“There is no way home, home is the way.” - Thich Nhat Hanh

#50
WhjteShadow

WhjteShadow

    Thượng úy

  • Phó Quản lý Toán Ứng dụ
  • 1323 Bài viết

Bài 33 (Thi thử THPT Chuyên ĐH Vinh lần 2-2010)
Cho các số thực không âm $x,y,z$ thỏa mãn $xy+yz+zx=3$. Tìm giá trị nhỏ nhất của biểu thức
$A=x^{2}y^{3}+y^{2}z^{3}+z^{2}x^{3}+\left ( x-1 \right )^{2}+\left ( y-1 \right )^{2}+\left ( z-1 \right )^{2}$.

Ta sẽ chứng minh $A\geq 3$.Hay là:
$$x^{2}y^{3}+y^{2}z^{3}+z^{2}x^{3}+\left ( x-1 \right )^{2}+\left ( y-1 \right )^{2}+\left ( z-1 \right )^{2}\geq 3$$
$$\Leftrightarrow x^{2}y^{3}+y^{2}z^{3}+z^{2}x^{3}+x^2+y^2+z^2-2(x+y+z)\geq 0$$
Áp dụng bất đẳng thức $AM-GM$ ta có:$x^{2}y^{3}+x+1\geq 3xy\,,\,\,y^{2}z^{3}+y+1\geq 3yz\,,\,\,z^{2}x^{3}+z+1\geq 3zx$
Vậy nên:
$$x^{2}y^{3}+y^{2}z^{3}+z^{2}x^{3}+x+y+z+3\geq 3(xy+yz+zx)$$
$$\Leftrightarrow x^{2}y^{3}+y^{2}z^{3}+z^{2}x^{3}\geq 2(xy+yz+zx)-x-y-z$$
Cuối cùng ta chỉ phải chứng minh:
$$x^2+y^2+z^2-2(x+y+z)+2(xy+yz+zx)-x-y-z\geq 0$$
$$\Leftrightarrow (x+y+z)^2\geq 3(x+y+z)$$
$$\Leftrightarrow x+y+z\geq 3$$
Và điều này đúng the0 $AM-GM$ :$(x+y+z)^2\geq 3(xy+yz+zx)=9\to x+y+z\geq 3$
Vậy $Min_A=3$.Đẳng thức xảy ra khi $x=y=z=1$ $\blacksquare$

Bài viết đã được chỉnh sửa nội dung bởi WhjteShadow: 06-10-2012 - 12:45

“There is no way home, home is the way.” - Thich Nhat Hanh

#51
WhjteShadow

WhjteShadow

    Thượng úy

  • Phó Quản lý Toán Ứng dụ
  • 1323 Bài viết
Bài toán 36.
Ch0 các số thực không âm $a,b,c$ thỏa mãn $ab+bc+ca=1$.Tìm giá trị nhỏ nhất của biểu thức:
$$D=\frac{1}{a+b}+\frac{1}{b+c}+\frac{1}{c+a}-\frac{1}{a+b+c}$$
Bài toán 37.
Ch0 các số thực dương $x,y,z$ thỏa mãn $x+y+z=xyz$.Chứng minh rằng:
$$\frac{1}{\sqrt{1+x^2}}+\frac{1}{\sqrt{1+y^2}}+\frac{2}{\sqrt{1+z^2}}\leq \frac{9}{4}$$
“There is no way home, home is the way.” - Thich Nhat Hanh

#52
yeutoan11

yeutoan11

    Sĩ quan

  • Thành viên
  • 307 Bài viết
37)
Đặt $x=tanA;y=tanB;z=tanC$($A,B,C$ là số đó 3 góc của 1 tam giác )
BĐT cần CM :
$CosA+CosB+2CosC \leq \dfrac{9}{4}$
Ta có :
$CosA+CosB+2CosC=2Cos\frac{A+B}{2}Cos\frac{A-B}{2}+2-4Sin^2\frac{C}{2}\leq 2Cos\frac{A+B}{2}+2-4Sin^2\frac{C}{2}=2Sin\frac{C}{2}+2-4Sin^2\frac{C}{2}$
(Vì $Cos\frac{A-B}{2}\le 1$ và $Cos\frac{A+B}{2}>0$)
Vậy ta cần CM:
$Sin\frac{C}{2}+1-2Sin^2\frac{C}{2}\le \frac{9}{8}$
$\Leftrightarrow (Sin\frac{C}{2}-\frac{1}{4})^2\ge0$(Đúng)
Vậy Ta có ĐPCM
Dựng nước lấy việc học làm đầu. Muốn thịnh trị lấy nhân tài làm gốc.
NGUYỄN HUỆ
Nguyễn Trần Huy
Tự hào là thành viên VMF

#53
lehoanghiep

lehoanghiep

    Trung sĩ

  • Thành viên
  • 196 Bài viết
Bài 32 Vì $a,b\in \left [ 0;1 \right ]$ nên ta có $\frac{a^{3}+2}{b^{2}+1}\leq \frac{a^{2}+2}{b^{2}+1}=\left ( a^{2}+2 \right )\left ( 1-\frac{b^{2}}{b^{2}+1} \right )\leq \left ( a^{2}+2 \right )-\left ( a^{2}+2 \right )\frac{b^{2}}{2}=a^{2}-b^{2}+2-\frac{1}{2}a^{2}b^{2}$.
Tương tự, ta có $\frac{b^{3}+2}{c^{2}+1}\leq b^{2}-c^{2}+2-\frac{1}{2}b^{2}c^{2}$; $\frac{c^{3}+2}{a^{2}+1}\leq c^{2}-a^{2}+2-\frac{1}{2}c^{2}a^{2}$.
Suy ra $P\leq 6-\frac{1}{2}\left ( a^{2}b^{2}+b^{2}c^{2}+c^{2}a^{2} \right )\leq 6$.
Đẳng thức xảy ra khi $a=b=c=0$ hoặc hai trong ba số bằng $0$.

Bài viết đã được chỉnh sửa nội dung bởi lehoanghiep: 06-10-2012 - 16:31


#54
Katyusha

Katyusha

    Sĩ quan

  • Thành viên
  • 461 Bài viết

Khi đó $ \frac{a}{bc+1}+\frac{b}{ca+1}+\frac{c}{ab+1}+3$
$=a\left ( \frac{1}{bc+1}-\frac{c}{ca+b}-\frac{b}{ab+c} \right )+5$.

Bạn giải thích giúp mình đoạn này được không :(

#55
lehoanghiep

lehoanghiep

    Trung sĩ

  • Thành viên
  • 196 Bài viết

Bạn giải thích giúp mình đoạn này được không :(

Mình xin lỗi! Chỗ ấy là $\frac{a}{bc+1}+\frac{b}{ca+1}+\frac{c}{ab+1}\leq \frac{a}{bc+1}+\frac{b}{ca+b}+\frac{c}{ab+c}+3=a\left ( \frac{1}{ba+1}-\frac{c}{ca+b}-\frac{b}{ab+c} \right )+5$

#56
lehoanghiep

lehoanghiep

    Trung sĩ

  • Thành viên
  • 196 Bài viết
Bài 38 Cho $x,y,z$ là các số thực dương thỏa mãn $xy+yz+zx=3$. Chứng minh rằng
$\frac{1}{xyz}+\frac{4}{\left ( x+y \right )\left ( y+z \right )\left ( z+x \right )}\geq \frac{3}{2}$.

#57
WhjteShadow

WhjteShadow

    Thượng úy

  • Phó Quản lý Toán Ứng dụ
  • 1323 Bài viết

Bài 38 Cho $x,y,z$ là các số thực dương thỏa mãn $xy+yz+zx=3$. Chứng minh rằng
$\frac{1}{xyz}+\frac{4}{\left ( x+y \right )\left ( y+z \right )\left ( z+x \right )}\geq \frac{3}{2}$.

Do $xy+yz+zx=3$ nên $xyz\leq 1$.Từ đó suy ra $\frac{1}{2xyz}\geq \frac{1}{2}$.Vậy ta chỉ cần chứng minh:
$$\frac{1}{2xyz}+\frac{4}{(x+y)(y+z)(x+z)}\geq 1$$
Áp dụng bất đẳng thức AM-GM 2 số thì ta phải chứng minh:
$$\frac{1}{2xyz}.\frac{4}{(x+y)(y+z)(x+z)}\geq \frac{1}{4}$$
$$\Leftrightarrow \frac{8}{xyz(x+y)(y+z)(x+z)}\geq 1$$
$$\Leftrightarrow (xy+yz)(yz+zx)(zx+xy)\leq 8$$
Và điều này đúng the0 AM-GM 3 số:
$$(xy+yz)(yz+zx)(zx+xy)\leq \frac{(2xy+2yz+2zx)^3}{27}=\frac{8.27}{27}=8$$
Ta có ĐPCM.Dấu bằng xảy ra tại $x=y=z=1$
Bài 39.
Ch0 $x,y,z>0$ thỏa $xy+yz+zx=1$.Chứng minh rằng:
$$\frac{x}{y(1+x^2)}+\frac{y}{z(1+y^2)}+\frac{z}{x(1+z^2)}\geq \frac{9}{4}$$

Bài viết đã được chỉnh sửa nội dung bởi WhjteShadow: 08-10-2012 - 12:14

“There is no way home, home is the way.” - Thich Nhat Hanh

#58
25 minutes

25 minutes

    Thành viên nổi bật 2015

  • Hiệp sỹ
  • 2795 Bài viết
Bài 40:Trên Toán học Tuổi trẻ,có đăng nhưng k thấy bài?
Cho a,b,c dương thoả mãn a,b,c$\in \left [ \frac{1}{3},3 \right ]$
Tìm min của P=$\frac{a}{a+b}+\frac{b}{b+c}+\frac{c}{c+a}$?
Hãy theo đuổi đam mê, thành công sẽ theo đuổi bạn.



Thảo luận BĐT ôn thi Đại học tại đây


#59
Ispectorgadget

Ispectorgadget

    Nothing

  • Quản lý Toán Phổ thông
  • 2946 Bài viết

Bài 40:Trên Toán học Tuổi trẻ,có đăng nhưng k thấy bài?
Cho a,b,c dương thoả mãn a,b,c$\in \left [ \frac{1}{3},3 \right ]$
Tìm min của P=$\frac{a}{a+b}+\frac{b}{b+c}+\frac{c}{c+a}$?

Bài này có ở đây.

►|| The aim of life is self-development. To realize one's nature perfectly - that is what each of us is here for. ™ ♫


#60
lehoanghiep

lehoanghiep

    Trung sĩ

  • Thành viên
  • 196 Bài viết
Bài 41 Cho các số thực $x,y$ thỏa mãn $x+y-3=4\left ( \sqrt{x-3}+\sqrt{y+1} \right )$.
Tìm giá trị lớn nhất và giá trị nhỏ nhất của biểu thức $M=\frac{1}{x-2}+\frac{1}{y+2}$.

Bài viết đã được chỉnh sửa nội dung bởi lehoanghiep: 07-10-2012 - 18:28





1 người đang xem chủ đề

0 thành viên, 1 khách, 0 thành viên ẩn danh